LSAT and Law School Admissions Forum

Get expert LSAT preparation and law school admissions advice from PowerScore Test Preparation.

 reticulargirl
  • Posts: 5
  • Joined: May 28, 2018
|
#46309
Alex,

Thank you. I think I understand where I went wrong. The trouble is that this analytical process occurs so fast that if I've gone wrong I sometimes don't realize it until later. :/
 Alex Bodaken
PowerScore Staff
  • PowerScore Staff
  • Posts: 136
  • Joined: Feb 21, 2018
|
#46350
reticulargirl,

No worries! It is often hard to slow our brains down to pick our exactly what we are analyzing...but the more we can return to questions like these to understand our own thinking, the better equipped we are for next time. Keep it up!

Best,
Alex
 whardy21
  • Posts: 48
  • Joined: Sep 30, 2018
|
#65407
I got this one correct. However I did not spot the cause and effect In this problem. From reading this over I have:

Cause: Social Intertia :arrow: Effect: Determinant of Human Behavior

Find alternate cause for the stated effect would still be answer A. People believe that the thought of job loss that technological innovations propose causes their behavior toward resisting technological innovations. Please check my logic. I want to see if I am in the realm of answering the question this way.
 Brook Miscoski
PowerScore Staff
  • PowerScore Staff
  • Posts: 418
  • Joined: Sep 13, 2018
|
#65436
whardy,

The stimulus chooses between two possible causes--social inertia and the desire for comfort or safety. To weaken the stimulus, show that its choice was wrong--and that gets you (A). You would, as you indicate, still get (A) if you were simply looking for an alternative cause in the way that you describe. You're in the ballpark, and this is an easy or medium question. There may be some situations where it would be important for you to realize that the stimulus wasn't just arguing for one cause, but asserting that the other potential cause was not the cause. In this case, you got it right anyway, but I would strive for more precision.
 jackbenimble106
  • Posts: 10
  • Joined: Oct 07, 2020
|
#85811
Hi there!

I'm having trouble seeing how (C) doesn't weaken the conclusion.

If the conclusion states that social inertia is a more powerful determinant of human behavior than the desire for comfort and safety, and there are examples in the past of technology being embraced by workers that would make their work less miserable, how would that not be an example of when the desire for comfort and safety determines human behavior more so than social inertia, thereby undermining the conclusion?
User avatar
 Ryan Twomey
PowerScore Staff
  • PowerScore Staff
  • Posts: 141
  • Joined: Mar 04, 2021
|
#85839
Hey Jackbeninmle106,

So to start, answer choice C displays very very weak language with "some examples of technological innovation have been embraced by workers." Some only means at least one, so the answer choice is providing one example of technological innovation being embraced by workers. This does not do much to affect social inertia being a more powerful determinant than comfort or safety. If the conclusion said, social inertia is always a more powerful determinant than comfort or safety, then maybe that answer choice of "some" would carry more weight.

Whereas A provides us with an alternative cause to show that the phenomenon has nothing to do with social inertia, they are simply acting in their best interest in wanting to hold on to their job.

I hope this helps. Answer choices with the quantifier "some" often don't do a whole lot to our conclusion because of how weak they are unless our conclusion is very strong with an "all, always, every" quantifier. This is not to say those answers are always incorrect, but the weakness of the language makes them difficult to be correct unless our conclusion involves a 100 percent quantifier like the ones I mentioned above.

Good luck with your studies!!!

Best,
Ryan
 jackbenimble106
  • Posts: 10
  • Joined: Oct 07, 2020
|
#85850
That was very helpful, Ryan. Thank you!
User avatar
 yufenz
  • Posts: 3
  • Joined: Mar 26, 2023
|
#100674
Quick question for Answer C

Would C be a good answer if it is worded like this: "Almost all technological innovations are embraced by workers". Something that directly confronts/invalidates the premise.

Thanks
 Luke Haqq
PowerScore Staff
  • PowerScore Staff
  • Posts: 747
  • Joined: Apr 26, 2012
|
#100693
Hi yufenz!

Even with that wording, there still seems to be a problem with (C). Namely, it doesn't get to the reasons why workers embraced technological innovations in the past. Was it because other workers accepted them and they followed along? Because of a desire for comfort and safety?

Without knowing more, it's not clear how that answer choice would address the conclusion's comparative claim about cause and effect, specifically about one option over another being the cause of certain human behavior.

Get the most out of your LSAT Prep Plus subscription.

Analyze and track your performance with our Testing and Analytics Package.